Difference between revisions of "2017 AMC 8 Problems/Problem 4"

(Created page with "==Problem 4== When <math>0.000315</math> is multiplied by <math>7,928,564</math> the product is closest to which of the following? <math>\textbf{(A) }210\qquad\textbf{(B) }24...")
 
(Solution:)
Line 5: Line 5:
  
 
==Solution:==
 
==Solution:==
We can approximate <math>7,928,564</math> to <math>8,000,000,</math> and <math>0.000315</math> to <math>0.0003.</math> Multiplying the two yields <math>2400.</math> This gives our answer to be <math>\text{D)}2400.</math>
+
We can approximate <math>7,928,564</math> to <math>8,000,000,</math> and <math>0.000315</math> to <math>0.0003.</math> Multiplying the two yields <math>2400.</math> This gives our answer to be <math>\text{D)}</math> <math>2400.</math>

Revision as of 15:04, 22 November 2017

Problem 4

When $0.000315$ is multiplied by $7,928,564$ the product is closest to which of the following?

$\textbf{(A) }210\qquad\textbf{(B) }240\qquad\textbf{(C) }2100\qquad\textbf{(D) }2400\qquad\textbf{(E) }24000$

Solution:

We can approximate $7,928,564$ to $8,000,000,$ and $0.000315$ to $0.0003.$ Multiplying the two yields $2400.$ This gives our answer to be $\text{D)}$ $2400.$